LSAT and Law School Admissions Forum

Get expert LSAT preparation and law school admissions advice from PowerScore Test Preparation.

User avatar
 Dave Killoran
PowerScore Staff
  • PowerScore Staff
  • Posts: 5849
  • Joined: Mar 25, 2011
|
#59776
Complete Question Explanation
(The complete setup for this game can be found here: lsat/viewtopic.php?t=26553)

The correct answer choice is (C)

As usual, apply the List question technique. Answer choice (A) can be eliminated since bench 2 contains two majors, a violation of the rules (and, of course, bench 4 contains two nonmajors). This question is rather irritating because of the layout chosen by the test makers. It would have been much easier had they separated the benches with some space instead of lining them up. Answer choice (C) is correct.

Get the most out of your LSAT Prep Plus subscription.

Analyze and track your performance with our Testing and Analytics Package.